AAMC CBT8 and 8R OFFICIAL Q&A

This forum made possible through the generous support of SDN members, donors, and sponsors. Thank you.

Vihsadas

No summer
Moderator Emeritus
Lifetime Donor
10+ Year Member
5+ Year Member
15+ Year Member
Joined
Oct 17, 2007
Messages
5,474
Reaction score
56
This is the official Q&A thread for AAMC CBT8 and 8R.

Please post ONLY questions pertaining to AAMC CBT8 and 8R.
Out of respect for people who may not have completed the other exams, do not post questions or material from any other AAMC exam.

Please see this thread for the rules of order before you post.

Good luck on your MCAT!
 
BS:

95: Which would not cause rickets?

The passage specifically says that Vitamin D enhances the effect of parathyroid hormone. It also says that rickets is caused by insufficient vitamin D. That means in rickets, the effect of PTH is not enhanced. This would equal an apparent deficiency in PTH.

I thought roman numeral III was not correct, because complete inability of vitamin D to act on its receptors would probably be a lot more harmful than cause rickets.

I thought the ideal choice would be all three, but that wasn't there.
 
VR:

86: Mountain climber choosing another rope.

Why is this not C? The correct answer is A, but I didn't choose that one because it never specifies that a belief in the weakness of the rope is the ONLY reason, just a belief. But C claims that a belief in the weakness of the rope is PROBABLY NOT a reason for the changing of the rope. Where does this reasoning come from?

So the author rejects the behavioralist approach. He attributes that a decision is based on a multitude of factors (e.g. umbrella example). The question stem asks "with which of the following conclusion about this acceptance would the author be LEAST apt to agree?" The author is likely to agree with C, because it implies that other factors (outside the obvious one) come into play when making a decision. He is unlikely to agree with A because it reduces your action down to one motive (behavioralist thinking). Does that make sense? I know I basically reworded the AAMC explanation, but I was not sure how else to explain it. Perhaps you missed the word LEAST in the question stem?
 
BS:

95: Which would not cause rickets?

The passage specifically says that Vitamin D enhances the effect of parathyroid hormone. It also says that rickets is caused by insufficient vitamin D. That means in rickets, the effect of PTH is not enhanced. This would equal an apparent deficiency in PTH.

I thought roman numeral III was not correct, because complete inability of vitamin D to act on its receptors would probably be a lot more harmful than cause rickets.

I thought the ideal choice would be all three, but that wasn't there.

Yeah...its sucks starting a section with a tricky question. So remember that Vitamin D ENHANCES parathyroid hormone activity, but is especially important in absorbing calcium. In rickets, the ratio of mineral to organic matter is lower than normal IN BONES (you know this because it says "bone matrix" in the sentence above). If you wipe out parathyroid hormone activity, you can't break down the bone matrix. Thus, there will be a buildup of calcium mineral. Consequently, you would have a HIGHER mineral to organic matter ratio (which is the opposite of rickets). Therefore, roman numeral I is incorrect. Roman numeral II and III, functionally, mean the same thing: less calcium mineral for bone since normally working vitamin D works to absorb calcium from diet in intestine (thus giving it the opportunity to incorporate into the bone matrix). Therefore, because roman numerals II and III both mean calcium would not be able to be absorbed from intestine and incorporated into the bone matrix, bones would have a lower mineral to organic material ratio.=> Rickets! The primary function of vitamin D is on the intestine tissue...so its important to focus on that...getting hung up on its enhancing effect on PH will trip you up and cause you to doubt whether III is correct. But since II is definitely right and I is definitely wrong, your only option is to assume that III is correct.
 
At what concentration of free actin will the + end of the microfillament grow faster than the - end?
a) exactly at 1um
b) only between 1um and 4um
c) at any concentration greater than 1um
d) at any concentration

OK, so at first glance C (which is correct) seems obvious since it is at this point that the + end actually begins adding subunits and growing. BUT at concentrations lower than 1um the + end is losing subunits at a slower rate compared to the - end (e.g. at .5um the + end is losing about 1 subunit/msec whereas the - end is losing about 3 subunits/msec). Therefore, it seems that the + end is growing RELATIVE to the - end since it is losing subunits more slowly (the Y-axis is rate). Anyhow, I'm sure I read way too deeply into the question - I'm a HPHYS major so its no surprise that I was trying to make this MCAT physiology question more complex than it really was. But did anybody else also follow my reasoning? OR (and this would be better) can anybody tell me why my reasoning is wrong? Thanks!!
 
The + end is simply not growing. It is shrinking, in fact. Both ends are below 1 um. For something to grow faster than something else, it still needs to be growing in the first place.
 
The + end is simply not growing. It is shrinking, in fact. Both ends are below 1 um. For something to grow faster than something else, it still needs to be growing in the first place.

Yeah, but everything within the human body happens on relative scales. So while it is not actually growing, it is growing COMPARED to the - end. Its obvious that I overcomplicated the problem - the MCATs wouldn't really go into that much detail. But I remember having a question very similar to this on a HPHY 333 final and the prof basically said that its not the addition of individual subunits which determine growth (since they have very little effect in the grand scheme of things) but the relative changes in subunit concentrations. But I realize that if the MCATs wanted an answer in terms of the - end, they would have specified it. I should have stuck with my original answer lol.
 
Yeah, but everything within the human body happens on relative scales. So while it is not actually growing, it is growing COMPARED to the - end. Its obvious that I overcomplicated the problem - the MCATs wouldn't really go into that much detail. But I remember having a question very similar to this on a HPHY 333 final and the prof basically said that its not the addition of individual subunits which determine growth (since they have very little effect in the grand scheme of things) but the relative changes in subunit concentrations. But I realize that if the MCATs wanted an answer in terms of the - end, they would have specified it. I should have stuck with my original answer lol.

I think you're misunderstanding or mis-remembering what your prof said. If end A is shrinking at 1 subunit/minute and end B is shrinking at 3 subunits per minute, the molecule is NOT growing. End A is not growing and End B is not growing. A is shrinking more slowly than B but that doesn't mean A is growing.

If I'm getting shorter each year but my sister is getting even shorter than I am each year, I'm not growing more than she is. We're both shrinking.
 
I think you're misunderstanding or mis-remembering what your prof said. If end A is shrinking at 1 subunit/minute and end B is shrinking at 3 subunits per minute, the molecule is NOT growing. End A is not growing and End B is not growing. A is shrinking more slowly than B but that doesn't mean A is growing.

If I'm getting shorter each year but my sister is getting even shorter than I am each year, I'm not growing more than she is. We're both shrinking.

Yes, neither of you are growing, but you are growing COMPARED to your sister. For example, if you are both 60inches. Next year you will be 59 and she will be 57 (net difference 2). THe year after you will be 58 and she will be 54 (net difference 4). The year after, you will be 57 and she will be 51 (6). If you plot this on a graph (2,4,6) you have a linear trend which shows that you are growing in relation to your sister, but to everyone else you are both just getting shorter. Obviously, this analogy is a bit simplified, but thats what I mean when I say RELATIVE growth compared to the - end. Again, I know I'm wrong and misinterpreted the question stem, but there is relative growth as the net differences are getting larger. But yes, I understand what you are saying as thats exactly what I thought at first.
 
Yes, neither of you are growing, but you are growing COMPARED to your sister. For example, if you are both 60inches. Next year you will be 59 and she will be 57 (net difference 2). THe year after you will be 58 and she will be 54 (net difference 4). The year after, you will be 57 and she will be 51 (6). If you plot this on a graph (2,4,6) you have a linear trend which shows that you are growing in relation to your sister, but to everyone else you are both just getting shorter. Obviously, this analogy is a bit simplified, but thats what I mean when I say RELATIVE growth compared to the - end. Again, I know I'm wrong and misinterpreted the question stem, but there is relative growth as the net differences are getting larger. But yes, I understand what you are saying as thats exactly what I thought at first.

eh...w/e I'm abandoning this question. I have come to terms with myself and realize that I'm being stubborn. Any other day, I would have picked growth above 1um, but I'm just being stupid now. I suppose weeks of MCAT prep is frying my brain and causing me to overthink things. lol
 
Last edited:
eh...w/e I'm abandoning this question. I have come to terms with myself and realize that I'm being stubborn. Any other day, I would have picked growth above 1um, but I'm just being stupid now. I suppose weeks of MCAT prep is frying my brain and causing me to overthink things. lol

I got the question wrong too. We're just inclined to think that the MCAT is trying to trick us when it isn't!
 
I got the question wrong too. We're just inclined to think that the MCAT is trying to trick us when it isn't!

No kidding. With so many tricks on the test, I often find that I second-guess myself when I come across a question that seems too easy...
 
A few questions:

PS

12: B2 rotates an H nucleus through an angle of.

b) 90
c) 180

The fact that it said through an angle, not to an angle got me. Since B2 is supposed to be perpendicular to B1, when the nucleus is in resonance, B2 would be at an angle of 90 degrees relative to the nucleus, so I chose b. I mean it seems like a common sense answer, but did I read into it too much?

I also picked b, by the same reasoning. Why isn't b the answer?
 
I can't understand the explanation for following three questions (I have attached the accompanying passage information):
Reaction 1 is an effective means of decreaing the hardness of acidic water sample because it:
A- increases the pH and decreases the solubility of CaCO3.
B- decreases the pH and decreases the solubility of CaCO3.
C- increases the pH and converts HCO3- to water.
D- decreases the pH and removes Ca2+ from solution.
I picked D. My thinking was that if Ca+ is being removed, equilibrium would shift to the left, increasing the amount of HCO3-, and thus decreasing pH. The correct answer is A.
What is the effect of a high level of atmosphere CO2 on the hardness of water in limestone regions?
A- It would lower the pH and increase the solubility of CaCO3.
B- It would raise the pH and increase the solubility of CaCO3.
C- It would lower the pH and decrease the solubility of CaCO3.
D- It would raise the pH and decrease the solubility of CaCO3.
For this, I got the right answer. But I just can't understand their reasoning. HELP!

Thanks.

PS. the questions are from 8R.
 

Attachments

Reaction 1 is an effective means of decreaing the hardness of acidic water sample because it:
A- increases the pH and decreases the solubility of CaCO3.
B- decreases the pH and decreases the solubility of CaCO3.
C- increases the pH and converts HCO3- to water.
D- decreases the pH and removes Ca2+ from solution.
I picked D. My thinking was that if Ca+ is being removed, equilibrium would shift to the left, increasing the amount of HCO3-, and thus decreasing pH. The correct answer is A.

I did these passages as well. Okay, so the reaction is:

Ca2+ + 2HCO3- --> CaCO3 (s) + H2O + CO2 (g).

The passage says that CaCO2 is soluble in acidic solutions, which is responsible for water hardness, and that the reaction above is one way to "soften acidic groundwater" by boiling, which removes calcium ion (as in the reaction above). Therefore, the right side/forward reaction is what we want to soften the hardwater. The forward reaction increases the pH (because the H can come off the HCO3- on the left side of the reaction). It also decreases the solubility of CaCO3 because the passage says that the "high degree of solubility of CaCO3 in acidic solutions (due to conversion to soluble Ca(HCO3)2)..."
What is the effect of a high level of atmosphere CO2 on the hardness of water in limestone regions?
A- It would lower the pH and increase the solubility of CaCO3.
B- It would raise the pH and increase the solubility of CaCO3.
C- It would lower the pH and decrease the solubility of CaCO3.
D- It would raise the pH and decrease the solubility of CaCO3.
For this, I got the right answer. But I just can't understand their reasoning. HELP!

This is just Le Chatelier's principle in disguise. Take another look at the reaction:

Ca2+ + 2HCO3- --> CaCO3 (s) + H2O + CO2 (g).

The question: What is the effect of a high level of atmosphere CO2 on the hardness of water in limestone regions?

High CO2 would shift the reaction to the left, which would produce more HCO3-, which would LOWER the pH and increase the solubility of CaCO3 (because the passage says that "the high degree of solubility of CaCO3 in acidic solutions..."

Hope this helps.


 
I did these passages as well. Okay, so the reaction is:

Ca2+ + 2HCO3- --> CaCO3 (s) + H2O + CO2 (g).

The passage says that CaCO2 is soluble in acidic solutions, which is responsible for water hardness, and that the reaction above is one way to "soften acidic groundwater" by boiling, which removes calcium ion (as in the reaction above). Therefore, the right side/forward reaction is what we want to soften the hardwater. The forward reaction increases the pH (because the H can come off the HCO3- on the left side of the reaction). It also decreases the solubility of CaCO3 because the passage says that the "high degree of solubility of CaCO3 in acidic solutions (due to conversion to soluble Ca(HCO3)2)..."

Thanks for your reply. Your reasoning for the second question makes sense; the only part confusing me was the solubility of the CaCO3.

As for the first question, I'm still confused. If Ca2+ is being removed, wouldn't the equilibrium shift to the left, increasing the solubility of CaCO3. Shift to the left would also lead to a decrease in H+, since HCO3- are being produced. Thanks.
 
Thanks for your reply. Your reasoning for the second question makes sense; the only part confusing me was the solubility of the CaCO3.

As for the first question, I'm still confused. If Ca2+ is being removed, wouldn't the equilibrium shift to the left, increasing the solubility of CaCO3. Shift to the left would also lead to a decrease in H+, since HCO3- are being produced. Thanks.

Okay maybe this should help. In the passage there is a short description that describes the reaction. It reads as such:
One way to soften acidic ground water is simply boiling, which results in the removal of calcium ions.

Ca2+ + 2HCO3- ---> CO2 + H20 + CaCO3

NOW when we boil the reactants to get rid of the Ca2+ we want to push the reaction to the right and have it stay there. It will defeat the purpose if the reaction goes back to the left because then there will still be Ca2+ in solution. We want to get rid of Ca2+. But also notice that if we get rid of CA2+ we also have to get rid of the HCO3- (which is an acid). Getting rid of an acid should make the solution more basic hence increase the pH.
 
I also picked b, by the same reasoning. Why isn't b the answer?

This is how I reasoned it out. I am not sure if I am right in my way of thinking but this is what I understood from what I read.

It is stated in the passage that the moment is parallel and then becomes antiparallel (180 degrees) with respect to B1 as it rotates around B2. Now remember B1 and B2 are perpendicular to each other. So if if it makes a 180 degree angle with B1 it also has to make a 180 degree angle with B2.
180.jpg


From the picture. We start out with the state on the left. Now when the Moment rotates arounds b2 it will have to be rotated 180 degress so that the moment is antiparallel to B1.
 
If anyone knows the answer to the above for sure then please tell me if I had the right thing in mind.
 
BS #106: How would a biologically normal kidney resolve the problem of an excess of glucagon causing hypertension?
 
Btw, does anyone have the score conversion chart for the PS section? I got 61/77😡 and was wondering if that was good enough for an 11.
 
go to e-mcat.com

make an account and click on the section that says " how the mcat is scored"

there you can find a score conversion
 
Whenever an object floats on a liquid, we know the object's weight is counteracted by the buoyant force.

W = Fb
mg = p(fluid) x V(obj) x g

As you can see, g will cancel out on both sides and is thereby irrelevant.



Flying bird over wild pig, because it is essential for the bird to "not require large amounts of water" for excretion in order for the bird to weigh less and be able to fly.
Sidenote: I find these questions that come up on AAMC really random and unrelated to "real" BS content/material.



Yeah I missed this too. I thought since they said it occurred only in males, it would be a sex-linked gene.

However I think their point is that some physical phenotypic characteristic (like this dewlap thingy) can occur only in males, yet still be an autosomal gene present in males and females. The gene may be only EXPRESSED in males, explaining why you see its phenotype in the one gender. So basically without further info you can't decide (so correct choice= autosomal or sex-linked).

Why cant shark be right? I thought stuff that lives in salt water need to have very concentrated urine to concerve their internal water supply?

Also the reason why it could be on autosomal or sex chromosome is because it is asking for the location of the actual gene. It could be that something on the sex chromosome that controls the geneis missed up. For example most of the gene for penis formation is NOT on Y chromosome. Only a control.
 
There MUST be some formatting error with this question. What in the world is "q" referring to? It's not labeled in the graph. Can anyone verify that this was probably just a formatting error?
 
There MUST be some formatting error with this question. What in the world is "q" referring to? It's not labeled in the graph. Can anyone verify that this was probably just a formatting error?
sorry i only have the full length one and number 49 is a phase diagram problem for me. Can you post what passage this problem is on?
 
sorry i only have the full length one and number 49 is a phase diagram problem for me. Can you post what passage this problem is on?

it's a standalone, third from the end. has to do with light and angles of reflection and refraction. they give a diagram with light rays and say which of the following must be true? hopefully thats enough for you to find it.
 
it's a standalone, third from the end. has to do with light and angles of reflection and refraction. they give a diagram with light rays and say which of the following must be true? hopefully thats enough for you to find it.
i dont see any q on the problem just alpha and theta
 
also, question 16 on the PS, cbt, gives a graph of the concentration of S4O6 as a function of time. btw choices b and d, b has the conc. leveling off after 19 sec and d has the conc. decreasing after 19 sec. i choice b correctly, but am wondering why the mcat did no consider the REVERSE reaction, which would have decreased s406 over time? is it bc the reverse reaction would have taken a lot longer, and wouldn't have decreased at the same rate it increased (the magnitudes of the slopes are the same, but i guess the reverse reaction wouldnt happen as quickly, thermodynamically speaking) i just dont see how the concentration could remain CONSTANT after it reached its peak

and...if you wash something with "cold water," would that help recrystallize the product further or would that remove soluble impurities? the answer is remove soluble impurities--but im not sure why, since if you use hot water, the impurities will be more soluble in it (solubility incr. w/ temp). is it because the crystals would also be soluble in hot water and you dont want to dissolve those? if that's the answer, then i understand that, but i dont understand why washing something w/ cold water wouldnt also be to recrystallize the product, since cooling results in greater crystallization. this is in ref to 104 on aamc8. i thought a and b were both petty valid, and ended up choosing the wrong answer.
 
Last edited:
sorry, another question...ps 27: why would the mass of water in the tank not have an effect? the pressure matters...but as long as the pressure is the same at a certain point the container could be any size and the temperature of that point wouldnt change?
 
also, question 16 on the PS, cbt, gives a graph of the concentration of S4O6 as a function of time. btw choices b and d, b has the conc. leveling off after 19 sec and d has the conc. decreasing after 19 sec. i choice b correctly, but am wondering why the mcat did no consider the REVERSE reaction, which would have decreased s406 over time? is it bc the reverse reaction would have taken a lot longer, and wouldn't have decreased at the same rate it increased (the magnitudes of the slopes are the same, but i guess the reverse reaction wouldnt happen as quickly, thermodynamically speaking) i just dont see how the concentration could remain CONSTANT after it reached its peak


All REACTIONS MUST REACH EQUILIBRIUM!

so the concentration of S4O6 has to level off at some point.
 
true....HOWEVER in this situation the I2 is being constantly removed by starch, so shouldnt the reaction shift back to the left
 
BS #106: How would a biologically normal kidney resolve the problem of an excess of glucagon causing hypertension?

yea can someone explain BS #106? I don't get it at all:

Assuming Hypothesis B to be correct, which of the following endocrine disorders would cause hypertension that could NOT be rectified by physiologically normal kidneys?

A.) An excess of aldosterone
B.) An excess of glucagon
C.) A shortage of thyroxine
D.) A shortage of insulin

What's the thought process for this one
 
yea can someone explain BS #106? I don't get it at all:

Assuming Hypothesis B to be correct, which of the following endocrine disorders would cause hypertension that could NOT be rectified by physiologically normal kidneys?

A.) An excess of aldosterone
B.) An excess of glucagon
C.) A shortage of thyroxine
D.) A shortage of insulin

What's the thought process for this one

yeah i dont get this at all.. this is definitely a logic question. the idea behind this is that, if the hormone is acting on the kidney itself, the kidney will have no power to rectify the problem- this really should be in the VR.. so lame.. all the other choices do not directly have anything to do with the kidney, so leap of faith you have to make that is that since none of those horomones are acting on the kidney, the kidney will magically be able to rectify the situation...

once again, this is critical thinking at its best or worst depending on how you look at it.. i idiotically chose D because i thought if you didnt have enough isulin you'd have a lot of glucose in the blood which would lead to more BP... whatever..
 
yeah i dont get this at all.. this is definitely a logic question. the idea behind this is that, if the hormone is acting on the kidney itself, the kidney will have no power to rectify the problem- this really should be in the VR.. so lame.. all the other choices do not directly have anything to do with the kidney, so leap of faith you have to make that is that since none of those horomones are acting on the kidney, the kidney will magically be able to rectify the situation...

once again, this is critical thinking at its best or worst depending on how you look at it.. i idiotically chose D because i thought if you didnt have enough isulin you'd have a lot of glucose in the blood which would lead to more BP... whatever..

I used the same logic...

by the way does anybody have any good explanations or links to how diabetes symptoms affect kidney function?
 
Question about BS #112: The most effective way to remove triethylamine in the workup of an organic reaction would be to extract the rxn mixture with:
a. sodium bicarbonate
b. sodium bisulfite
c. sodium sulfate
d. hydrochloric acid <= correct answer

I got this question right by guessing b/c HCl was the only one I could vaguely make up a mechanism for, but I'm still a little confused (and the AAMC explanation didn't really help) Some posters above basically said just to know that HCl extracts amines - but can someone explain why a, b, and c won't work?
 
Question about BS #112: The most effective way to remove triethylamine in the workup of an organic reaction would be to extract the rxn mixture with:
a. sodium bicarbonate
b. sodium bisulfite
c. sodium sulfate
d. hydrochloric acid <= correct answer

I got this question right by guessing b/c HCl was the only one I could vaguely make up a mechanism for, but I'm still a little confused (and the AAMC explanation didn't really help) Some posters above basically said just to know that HCl extracts amines - but can someone explain why a, b, and c won't work?

Because A, b, C are not acid. You need an acid to protonate the amine. once the amine is protionated it has a net charge of +1. This makes it very water soluble.
 
This is from AAMC8 BS

Which of the following characteristics clearly marks fungi as eukaryotes?
A) they have cell walls
B) they contain ribosomes
C) they contain mitochondria
D) they exhibit sexual reproduction

Answer is C, but why not D??

Since when do prokaryotes reproduce sexually????????????
 
Last edited:
This is from AAMC8 BS

Which of the following characteristics clearly marks fungi as eukaryotes?
A) they have cell walls
B) they contain ribosomes
C) they contain mitochondria
D) they exhibit sexual reproduction

Answer is C, but why not D??

Since when do prokaryotes reproduce sexually????????????
 
Conjugation, Transduction, Transformation...

REVIEW NOW!

Well, i dunno about the second two. But conjugation involves a SEX pili... as for the other 2, they are all form of genetic recombination... sex...
 
Last edited:
This is from AAMC8 BS

Which of the following characteristics clearly marks fungi as eukaryotes?
A) they have cell walls
B) they contain ribosomes
C) they contain mitochondria
D) they exhibit sexual reproduction

Answer is C, but why not D??

Since when do prokaryotes reproduce sexually????????????

bacterial pseudo "sex" is called conjugation
 
Why do calcium supplements often include vitamin D? A
) Vitamin D is needed to prevent rickets. B
) The activated form of vitamin D stimulates the absorption of calcium into the blood.
Activated vitamin D acts on the small intestine to stimulate the absorption of calcium into the bloodstream. The inclusion of vitamin D in calcium supplements would ensure that vitamin D is present in the body to help promote this absorption. Thus, B is the best answer.


C
) The activated form of vitamin D enhances the action of calcitonin. D
) The activated form of vitamin D enhances the uptake of calcium by bone tissue.
I understand why B is a correct answer, but looking at the passage: "Vitamin D...acts on the small intestine to stimulate absorption of calcium and also on bone tissue". So I took this to mean it increased Ca absorption in the intestines and in bones. Given this, why would D we wrong especially considering that one would take Ca pills not to have a high blood Ca level, but to build bone.
 
#102 in Bio..

Q: Which semicarbazone is the product of thermodynamic control?

A: 2-Furaldeyhde's because it is produced under equilibrium conditions and is more stable than the other product.

I understand the reasoning of the answer, saying that the higher the melting point, the greater the stability. What I don't understand is what the "under equilibrium conditions" has anything to do with anything. Not really seeing the connection with equilibrium conditions vs. non equilibrium conditions I guess..
 
#102 in Bio..

Q: Which semicarbazone is the product of thermodynamic control?

A: 2-Furaldeyhde's because it is produced under equilibrium conditions and is more stable than the other product.

I understand the reasoning of the answer, saying that the higher the melting point, the greater the stability. What I don't understand is what the "under equilibrium conditions" has anything to do with anything. Not really seeing the connection with equilibrium conditions vs. non equilibrium conditions I guess..

i think the keypoint is that it's "more stable than the other product"
 
Top